Clase 12 Soluciones RD Sharma – Capítulo 18 Máximos y mínimos – Ejercicio 18.5 | Serie 1

Pregunta 1. Determinar dos números positivos cuya suma sea 15 y la suma de cuyos cuadrados sea mínima.

Solución:

Supongamos que los dos números positivos son x e y, 

Y se da que x + y = 15 …..(i)

Entonces, sea P = x 2 + y 2                     …..(ii)

De la ecuación (i) y (ii), obtenemos

P = x 2 + (15 – x) 2

Al derivar wrt x, obtenemos

dP/dx = 2x + 2(15 – x)(-1)

= 2x -30 +2x

= 4x ​​-30

Para máximos y mínimos.

Poner dP/dx = 0

⇒ 4x – 30 = 0

⇒ x = 15/2

Ya que, d 2 P/dx 2 = 4 > 0

Entonces, x = 15/2 es el punto de los mínimos locales,

De la ecuación (i), obtenemos

 y = 15 – 15/2 = 15/2

Entonces, los dos números positivos son 15/2, 15/2.

Pregunta 2. Divide 64 en dos partes de manera que la suma de los cubos de dos partes sea mínima.

Solución:

Supongamos que 64 se divide en dos partes, es decir, x e y 

Entonces, x + y = 64 …..(i)

Sea P = x 3 + y 3               ………(ii)

De la ecuación (i) y (ii), obtenemos

P = x 3 + (64 – x) 3

Al derivar wrt x, obtenemos

dP/dx = 3x 2 + 3(64 – x) 2 × (-1)

= 3x 2 – 3(4096 – 128x + x 2 )

= -3 (4096 – 128x)

Para máximos y mínimos.

Poner dP/dx = 0 

⇒ -3(4096 – 128x) = 0

⇒ x = 32

Ahora,

d 2 s/dx 2 = 384 > 0

Entonces, x=32 es el punto de los máximos locales.

Por lo tanto, el 64 se divide en dos partes iguales que es (32, 32)

Pregunta 3. ¿Cómo debemos elegir dos números, cada uno mayor o igual a -2, cuya suma sea 1/2 para que la suma del primero y el cubo del segundo sea mínima?

Solución:

Supongamos que x e y son los dos números, tales que x, y ≥ -2 y

x + y = 1/2 ……(i)

Entonces, sea P = x + y 3             …….(ii)

De la ecuación (i) y (ii), obtenemos

P = x + (1/2 – x) 3

Al derivar wrt x, obtenemos

dP/dx = 1 + 3(1/2 – x) 2 × (-1)

= 1 – 3(1/4 – x + x 2 )

= 1/4 +3x -3x 2

Para máximo y mínimo,

Poner dP/dx = 0

⇒ 1/4 + 3x – 3x 2 = 0

⇒ 1 + 12x – 12x 2 = 0

⇒ 12x 2 – 12x – 1 = 0

⇒ x=\frac{12±\sqrt{144+48}}{24}

⇒ x = 1/2 ± (8√3/24)

⇒ x = 1/2 ± (1/√3)

⇒ x = {1/2 – (1/√3)}, {1/2 + (1/√3)}

Ahora,

d 2 P/dx 2 = 3 – 6x

Entonces, en x =1/2 – (1/√3), d 2 P/dx 2 = 3(1 – 2(1/2 – 1/√3))

= 3(+2/√3) = 2√3 > 0

Por lo tanto, x = 1/2 – 1/√3 es el punto de mínimos locales

De la ecuación (i), obtenemos

y = 1/2 – (1/2 – 1/√3) = 1/√3

Entonces, los números son (1/2 – 1/√3) y 1/√3

Pregunta 4. Dividir 15 en dos partes de manera que el cuadrado de una multiplicado por el cubo de la otra sea mínimo.

Solución:

Supongamos que 15 se divide en dos partes, es decir, x e y 

Entonces, x + y = 15

Además, P = x 2 y 3

De la ecuación (i) y (ii), obtenemos

P = x 2 (15 – x) 3

Al derivar wrt x, obtenemos

dP/dx = 2x(15 – x) 3 – 3x 2 (15 – x) 2

= (15 – x) 2 [30x – 2x 2 – 3x 2 ]

= 5x(15 – x) 2 (6 – x)

Para máximos y mínimos,

Poner dP/dx = 0

⇒ 15(15 – x) 2 (6 – x) = 0

⇒ x = 0, 15, 6

Ahora,

Entonces, d 2 P/dx 2 = 5(15 – x) 2 (6 – x) – 5x × 2(15 – x)(6 – x) – 5x(15 – x) 2

En x = 0, d 2 P/dx 2 = 1125 > 0

Entonces, x = 0 es el punto de mínimos locales

En x = 15, d 2 P/dx 2 = 0

Entonces, x = 15 es un punto de inflexión.

En x = 6, d 2 P/dx 2 = -2430 < 0

Entonces, x = 6 es el punto de los máximos locales

Entonces, el 15 se divide en dos partes que son 6 y 9.

Pregunta 5. De todas las latas cilíndricas cerradas (circulares rectas), que encierran un volumen dado de 100 cm 3 ¿cuál tiene la superficie mínima?

Solución:

Supongamos que r es el radio del cilindro y h la altura del cilindro

Entonces, el volumen del cilindro es de 100 cm 3

es decir, V = πr 2 h = 100

h = 100/πr 2 ……(i)

Ahora encontramos que el área de la superficie del cilindro es

A = 2πr 2 + 2πrh 

= 2πr 2 + 200/r

Al diferenciar wrt r, obtenemos

dA/dr = 4πr – 200/r 2

⇒ d 2 A/dr 2 = 4π + 400/r 3

Para máximos y mínimos,

dA/dr = 0 

⇒ 4πr = 200/ r2

⇒ r3 = 200/4π = 50/π

r = (50/π) 1/3

Entonces, cuando r = (50/π) 1/3 , d 2 s/dr 2 > 0

Por lo tanto, de la prueba de la segunda derivada, el área superficial es la mínima 

cuando el radio del cilindro es (50/π) 1/3 cm

Ahora ponga el valor de r en la ecuación (i), obtenemos

h = 100 / π(50/π) 1/3 = (2×50)/(50 2/3 π 1-2/3 ) = 2(50/π) 1/3

Pregunta 6. Una viga se apoya en los dos extremos y se carga uniformemente. El momento de flexión M a una distancia x de un extremo viene dado por

(1) M = (WL/2)x – (w/2)x 2

(ii) M = Wx/3 – (W/3) (x 3 /L 2 )

Encuentre el punto en el que M es máximo en cada caso.

Solución:

(i) M = (WL/2)x – (w/2)x 2

Al derivar wrt x, obtenemos

dM/dx = WL/2 – Wx

Para máximos y mínimos,

Poner dM/dx = 0 

WL/2 – Wx = 0   

x = L/2

Ahora, d 2 M/dx 2 = -W < 0

Entonces, x = L/2 es el punto de los máximos locales.

Por lo tanto, M es máximo cuando x = L/2

(ii) M = Wx/3 – (W/3)(x 3 /L 2 )

Al derivar wrt x, obtenemos

dM/dx = W/3 – Wx 2 /L 2

Para máximos y mínimos,

Poner dM/dx = 0 

W/3 – Wx 2 /L 2 = 0  

x = ± L/√3

Ahora, d 2 M/dx 2 = – 2xW/L 2

Entonces, en x = L/√3, ⇒ d 2 M/dx 2 =-2W/√3L < 0 (para el valor máximo)

en x = -L/√3, ⇒ d 2 M/dx 2 = 2W/√3L > 0 (para valor mínimo)

Por lo tanto, M es máximo cuando x = L/√3

Pregunta 7. Un alambre de 28 m de longitud debe cortarse en dos pedazos. Una de las piezas se va a hacer cuadrada y la otra en forma de círculo. ¿Cuáles deberían ser las longitudes de las dos piezas para que el área combinada del círculo y el cuadrado sea mínima?  

Solución:

Supongamos que lm es el trozo de longitud cortado del alambre dado para formar un cuadrado.

y el otro trozo de alambre que se usa para crear un círculo tiene una longitud de (28-l) m.

Entonces, el lado del cuadrado = l/4

Ahora, consideremos que el radio del círculo es r. 

Entonces, 2πr = 28 – l ⇒ r = (1/2π)(28 – l)

Ahora encontramos el área combinada del cuadrado y el círculo.

A = l 2 /16 + π[(1/2π)(28 – l)] 2

= l 2 /16 + 1/4π (28 – l) 2

Al diferenciar wrt l, obtenemos

dA/dl = 2l/16 + (2/4π)(28 – l)(-1)

= l/8 – (1/2π)(28 – l)

d 2 A/dl 2 = l/8 + (1/2π) > 0

Para máximos y mínimos,

Poner dA/dl = 0 

⇒ l/8 – (1/2π)(28 – l) = 0

⇒ {πl – 4(28 – l)}8π = 0

⇒ (π + 4)l – 112 = 0

⇒ l = 112/(π + 4)

Entonces, en l = 112/(π + 4), d 2 A/dl 2 > 0

Por lo tanto, usando la prueba de la segunda derivada, el área es el mínimo cuando l = 112/(π + 4)

Entonces, la longitud de las dos piezas de alambre es 112/(π + 4) y 28π/(π + 4) cm.

Pregunta 8. Un alambre de 20 m de longitud debe cortarse en dos pedazos. Una de las piezas se doblará en forma de cuadrado y la otra en forma de triángulo equilátero. ¿Dónde se debe cortar el alambre para que la suma de las áreas del cuadrado y del triángulo sea mínima?  

Solución:

Según la pregunta

La longitud del cable es de 20 m.

y el alambre cortado en dos piezas x e y. Entonces, se usa alambre de longitud x para hacer un cuadrado y

El alambre de longitud y se usa para hacer un triángulo.

Ahora.

x + y = 20 …..(yo)

x = 4l y y = 3a

Entonces, A = suma del área de un cuadrado y un triángulo

A = l 2 + √3/4a 2         ……(ii)

Tenemos, 4l + 3a =20

4l = 20 – 3a

l = (20 – 3a) / 4

De la ecuación (i), tenemos,

A = (20 – 3a) 2 /4 + √3/4a 2

Al derivar wrt a, obtenemos

dA/da = 2{(20 – 3a)/4}(-3/4) + 2a × √3/4

Para máximos y mínimos,

Poner dA/da = 0

⇒ 2 {(20 – 3a)/4}(-3/4) + 2a × √3/4 = 0

⇒ -3(20 – 3a) + 4a√3 = 0

⇒ -60 + 9a + 4a√3 = 0

⇒ 9a + 4a√3 = 60

⇒ a(9 + 4√3) = 60

⇒ a = 60/(9 + 4√3)

Nuevamente diferenciando wrt a, obtenemos

d 2 s/da 2 = (9 + 4√3)/8 > 0

Entonces, la suma de las áreas del cuadrado y del triángulo es mínima cuando a = 60 / (9 + 4√3)

Entonces l = (20 – 3a)/4

⇒ yo = \frac{20-3(60/(9+4√3))}{4}

⇒ l = (180 + 80√3 – 180)/{4(9 + 4√3)}

⇒ l = 20√3/(9 + 4√3)

Pregunta 9. Dada la suma de los perímetros de un cuadrado y un círculo, demuestre que la suma de sus áreas es mínima cuando un lado del cuadrado es igual al diámetro del círculo.

Solución:

Supongamos que el radio del círculo es r 

Tenemos,

2πr + 4a = k (k es constante)

a = (k – 2πr)/4

Ahora encontramos la suma de las áreas del círculo y el cuadrado:

A = πr 2 + a 2 = πr 2 + (k – 2πr) 2/16

Al diferenciar wrt r, obtenemos

dA/dr = 2πr + 2(k – 2πr)(-2π)/16 

= 2πr-π(k – 2πr)/4

Para máximos y mínimos,

Ponga, dA/dr = 0

2πr = π(k – 2πr)/4

8r = k – 2πr

r = k / (8 + 2π)= k / 2(4 + π)

Ahora, d 2 A/dr 2 = 2π + π 2 /2 > 0

Entonces, en r = k / 2(4 + π), d 2 A/dr 2 > 0

Por lo tanto, la suma de las áreas mínimas cuando r = k / 2(4 + π)

Entonces, un = \frac{k-2π[\frac{k}{2}(4 + π)]}{4}

= 2r 

Por lo tanto probado

Pregunta 10. Encuentra el área más grande posible de un triángulo rectángulo cuya hipotenusa mide 5 cm de largo.

Solución:

Supongamos que PQR es un triángulo rectángulo, 

Entonces, la hipotenusa h = PR = 5 cm.

Ahora, supongamos que a y b son los lados restantes del triángulo. 

Entonces, a 2 + b 2 = 25 ……(i)

Ahora encontramos el área de PQR = 1/2 QR × PQ

A = 1/2 ab ……(ii)

De la ecuación (i) y (ii), obtenemos

⇒ UN = 1/2 x √(25 – un 2 )

Al derivar wrt a, obtenemos

dA/da = \frac{1}{2}[\frac{\sqrt{25-a^2}-2a^2}{2\sqrt{25-a^2}}]

\frac{1}{2}[\frac{25-a^2-a^2}{\sqrt{25-a^2}}]

\frac{1}{2}[\frac{25-2a^2}{\sqrt{25-a^2}}]

Para máximos y mínimos,

Poner dA/da = 0

⇒ \frac{1}{2}[\frac{25-2a^2}{\sqrt{25-a^2}}] =0

⇒ a = 5/√2

Ahora,

d 2 A/d 2 a = \frac{1}{2}[\frac{{\sqrt{25-a^2} (-4a)+\frac{(25-2a^2)2a}{2\sqrt{25-a^2}}}}{25-a^2}]

En a = 5√2, d 2 s/d 2\frac{1}{2}[\frac{\frac{-25}{√2}×5√2+0}{\frac{25}{2}}]

= – 5/2 < 0

Entonces, x = 5/√2 es un punto máximo local,

Por lo tanto, el área más grande posible del triángulo

= 1/2 × (5/√2) × (5/√2) = 25/4 unidades cuadradas

Pregunta 11. Dos lados de un triángulo tienen longitudes ‘a’ y ‘b’ y el ángulo entre ellos es θ. ¿Qué valor de θ maximizará el área del triángulo? Encuentre también el área máxima del triángulo.

Solución:

Supongamos que ABC es un triángulo tal que AB = a, BC = b y ∠ABC = θ

y AD en perpendicular a BC.

BD = asenθ

Entonces, el área de △ABC = 1/2 × BC × AD

⇒ A = 1/2 × b × a × sinθ

Al diferenciar wrt θ, obtenemos

dA/dθ = 1/2 × abcosθ

Para máximos y mínimos,

Poner dA/dθ = 0

⇒ 1/2 × abcosθ = 0

⇒ cosθ = 0

⇒ θ = π/2

Ahora, d 2 A/dθ 2 = -1/2 ab senθ

En θ = π/2, d 2 A/dθ 2 = -1/2ab < 0

Entonces, θ = π/2, es el punto de los máximos locales

Por lo tanto, el área máxima del triángulo ABC es 1/2 × absin(π/2) = 1/2 ab.

Pregunta 12. Una pieza cuadrada de hojalata de 18 cm de lado se va a convertir en una caja sin tapa cortando un cuadrado de cada esquina y doblando las solapas para formar una caja. ¿Cuál debe ser el lado del cuadrado a cortar para que el volumen de la caja sea máximo? Además, encuentre este volumen máximo.  

Solución:

Supongamos que x cm es el lado del cuadrado a cortar.

Ahora, el largo y el ancho de la caja serán (18 – 2x) cm cada uno y 

los x cm ser la altura de la caja.

Entonces, el volumen de la caja es 

V(x) = x(18 – 2x) 2

Al derivar wrt x, obtenemos

V'(x) = (18 – 2x) 2 – 4x(18-2x)

= (18 – 2x)[18 – 2x – 4x]

= (18 – 2x)(18 – 6x)

= 6 × 2(9 – x)(3 – x)

= 12(9 – x)(3 – x)

Nuevamente al diferenciar wrt x, obtenemos

V”(x) = 12 [-(9 – x) – (3 – x)]

= -12 (9 – x + 3 – x)

= -12 (12 – 2x)

= -24 (6 – x)

Para máximos y mínimos,

Poner V'(x) = 0

12(9 – x)(3 – x) = 0

x = 9, 3

Cuando x = 9 el largo y el ancho de la caja se vuelven cero.

Entonces, x ≠ 9

Cuando x = 3, V”(x) = -24 (6 – x) = -72 < 0

Entonces, x = 3 es el punto de máximos

Por lo tanto, el volumen máximo es V x = 3 = 3(18 – 2 × 3) 2

⇒ V = 3 × 12 2

⇒ V = 3 × 144

⇒ V = 432 cm3 

Pregunta 13. Una lámina rectangular de hojalata de 45 cm por 24 cm se va a convertir en una caja sin tapa, cortando cuadrados de cada esquina y doblando las solapas. ¿Cuál debe ser el lado del cuadrado a cortar para que el volumen de la caja sea el máximo posible?

Solución:

Supongamos que x cm es el lado del cuadrado a cortar.

Entonces, la altura de la caja = x, 

la longitud de la caja = 45 – 2x, 

y el aliento de la caja = 24 – 2x.

Entonces, el volumen de la caja es 

V(x) = x (45 – 2x)(24 – 2x)

= x (1080 – 90x – 48x + 4x 2 )

= 4x ​​3 – 138x 2 + 1080x

Al derivar wrt x, obtenemos

V'(x)= 12x 2 – 276x + 1080

= 12(x 2 – 23x + 90)

= 12(x-18) (x-5)

Nuevamente al diferenciar wrt x, obtenemos

V”(x) = 24x – 276 = 12 (2x – 23)

Para máximos y mínimos,

Poner V'(x) = 0

4x 3 – 138x 2 + 1080x = 0

x(x-18)-5(x-18) = 0

(x-5)(x-18) = 0

Entonces, x = 18 y x = 5

cuando x = 18 no es posible cortar un cuadrado de 18 cm de lado de cada esquina de la hoja rectangular. 

Entonces, x ≠ 18

Cuando x = 5, V”(5) = 12 (10 – 23) = 12(-13) = -156 < 0

Entonces, x = 5 es el punto de máximos.

Por tanto, el volumen de la caja es máximo cuando x = 5.

Pregunta 14. Se va a construir un tanque con base rectangular y lados rectangulares abiertos en la parte superior de modo que su profundidad sea de 2 my el volumen de 8 m 3 . Si la construcción del tanque cuesta ₹ 70 por metro cuadrado para la base y ₹ 45 por metro cuadrado para los lados, ¿cuál es el costo del tanque menos costoso?

Solución:

Consideremos que la longitud, la anchura y la altura del tanque sean l, b y h

Según la pregunta

La altura del tanque es 2 y el volumen es 8m 3

Entonces, el volumen del tanque es  

V = l × b × h 

8 = l × b × 2

libras = 4  

⇒ b = 4/l ….(yo)

Ahora, encontramos el área de la base = lb = 4

y el área de las cuatro paredes (A) = 2h (l + b)

A = 4 (l + l/4)

Al diferenciar wrt l, obtenemos

⇒ dA/dl = 4 (l – 4/l 2 )

Nuevamente diferenciando wrt l, obtenemos

 d 2 A/dl 2 = 32/l 3

Para máximos y mínimos,

Poner dA/dl = 0

⇒ l – 4/l 2 = 0 

⇒ l 2 = 4

⇒ l = ±2

Como sabemos que la longitud no puede ser negativa. Entonces, l ≠ 2

Cuando l = 2, d 2 A/dl 2 = 32/8 = 4 > 0

Entonces, l = 2 es el punto de mínimos.

Ahora pon el valor de l = 2 cm en la eq(i)

b = 4/l = 4/2 = 2

Entonces, l = segundo = h = 2

Por lo tanto, el área es el mínimo cuando l = 2.

Entonces, el costo de construir la base = 70 × (lb) = 70 × 4 = 280

Costo de construir las paredes = 2h (l + b) × 45 = 90 × 2 × (2 + 2) = 8 × 90 = 720

Por lo tanto, el costo total = 280 + 720 = 1000

Pregunta 15. Una ventana en forma de rectángulo está coronada por una abertura semicircular. El perímetro total de la ventana es de 10 m. Encuentre las dimensiones de la parte rectangular de la ventana para admitir la máxima luz a través de toda la abertura.

Solución:

Supongamos que x e y son el largo y el ancho del rectángulo. 

Entonces el radio de la abertura semicircular = x/2

De la pregunta se da que el perímetro de la ventana es de 10 m.

Entonces, x + 2y + πx/2 = 10

⇒ x(1 + π/2) + 2y = 10

⇒ 2y = 10 – x(1 + π/2)

⇒ y = 5 – x(1/2 + π/4)

Ahora, el área de la ventana es

A = xy + (π/2)(x/2) 2

= x [5 – x(1/2 + π/4)] + (π/8)x 2

= 5x – x 2 (1/2 + π/4) + (π/8)x 2

Al derivar wrt x, obtenemos

dA/dx = 5 – 2x(1/2 + π/4) + (π/4)x

= 5 – x(1 + π/2) + (π/4)x

Nuevamente diferenciando wrt x, obtenemos

d 2 A/dx 2 = -(1 + π/2) + π/4 = -1 – π/4

Para máximos y mínimos,

Poner dA/dx = 0

⇒ 5 – x(1 + π/2) + (π/4)x = 0

⇒ 5 – x – (π/4)x = 0

⇒ x (1 + π/4) = 5

⇒ x = 5/(1 + π/4) = 20/(π + 4)

Entonces, cuando x = 20/(π + 4), d 2 A/dx 2 < 0

Entonces, el área es máxima cuando la longitud (x) = 20/(π + 4)

Ahora,

y = 5 – 20/(π + 4){(2 + π)/4} = 5 – 5(2 + π)/(π + 4) = 10/(π + 4) m 

Por lo tanto, la longitud del rectángulo es 20/(π + 4) my el ancho es 10/(π + 4) m.

Publicación traducida automáticamente

Artículo escrito por rahulsharma1771996 y traducido por Barcelona Geeks. The original can be accessed here. Licence: CCBY-SA

Deja una respuesta

Tu dirección de correo electrónico no será publicada. Los campos obligatorios están marcados con *